Convergenza quasi certa per una serie di Bernoulli

Messaggioda feddy » 12/10/2019, 22:51

Ciao a tutti, mi sono bloccato su questo esercizio che credevo facile, eppure ora per me non lo è :?

Sia $( \xi_i)_{i \geq 1}$ una sequenza di variabili aleatorie, dove per ciascuna $\xi$ si ha $P(\xi=+1)=P(\xi=-1)=\frac{1}{2}$.
Si consideri la serie
$\sum_{n \geq 1} \frac{\xi_n}{n}$


Si mostri che questa converge quasi certamente


Pensavo di applicare solamente la definizione di convergenza quasi certa, ossia provare che $\lim_m P(|X_n-X|< \varepsilon \text{ per ogni }n \geq m)=1$ ma non sono riuscito a cavarci molto. Infatti avrei

$P(|\sum_{n \geq 1} \frac{\xi_n}{n}|< \varepsilon)$, e non saprei proprio come gestire questo termine.
Avatar utente
feddy
Moderatore
Moderatore
 
Messaggio: 2585 di 5934
Iscritto il: 26/06/2016, 00:25
Località: SISSA

Re: Convergenza quasi certa per una serie di Bernoulli

Messaggioda tommik » 13/10/2019, 08:20

Le variabili della sucessione $xi _n/n$ si distribuiscono così:

$Y_n=xi _n/n={{: ( -1/n ,;1/2 ),(+ 1/n , ;1/2 ) :}$

...e la successione converge quasi certamente a zero

Dimostrazione:
Testo nascosto, fai click qui per vederlo
Applico il primo lemma di Borel Cantelli:

$sum_(n=1)^(+oo) mathbb{P}[|Y_n|>epsilon]<=sum_(n=1)^(floor(1/epsilon)) 1=floor(1/epsilon)<+oo$

dato che $epsilon$ è fissato




Quindi per il teorema di conservazione della continuità1 (Continuous Mapping Theorem) converge a zero qc anche la somma

Note

  1. ipotizzando che le variabili siano indipendenti e considerando che
    ${{: ( Sigma_n mathbb{E}[Y_n]=0 ),( Sigma_n mathbb{V}[Y_n]=Sigma_n1/n^2<+oo ) :}$
tommik
Moderatore
Moderatore
 
Messaggio: 5069 di 11278
Iscritto il: 23/04/2015, 13:13
Località: Cassano Magnago

Re: Convergenza quasi certa per una serie di Bernoulli

Messaggioda feddy » 13/10/2019, 09:14

Molte grazie tommik per la risposta. Purtroppo però non riesco a capire la tua applicazione del lemma di Borel - Cantelli (che non conoscevo). In particolare, non capisco come hai maggiorato la prima sommatoria.

Correggimi se sbaglio: tu hai mostrato che $\sum_{n} P(|Y_n|> \varepsilon) < + \infty$, da cui segue che $P(\lim"sup"_n X_n )=0$, e questo mostra la convergenza quasi certa, giusto?
Avatar utente
feddy
Moderatore
Moderatore
 
Messaggio: 2586 di 5934
Iscritto il: 26/06/2016, 00:25
Località: SISSA

Re: Convergenza quasi certa per una serie di Bernoulli

Messaggioda feddy » 13/10/2019, 15:03

Ottimo, ora mi torna tutto. Mi prenderò il tempo necessario per fare altri esercizi sulla convergenza q.c., visto che ultimamente la sto usando in modo massiccio. Ti ringrazio ancora per la pazienza e la spiegazione.

Ne approfitto, per chiederti delucidazioni anche sulla convergenza in $L^1$ della sommatoria. Sostanzialmente pensavo di fare lo "stesso" ragionamento sopra, ossia notare che ($Y_n$ distribuita come nei post sopra) $E[|Y_n|]=\frac{1}{n} \frac{1}{2} + \frac{1}{n} \frac{1}{2} = \frac{1}{n}$ q.o., da cui segue che ogni variabile $Y_n$ converge a $0$ in $L^1$, perciò anche la loro somma farà lo stesso.
Avatar utente
feddy
Moderatore
Moderatore
 
Messaggio: 2587 di 5934
Iscritto il: 26/06/2016, 00:25
Località: SISSA

Re: Convergenza quasi certa per una serie di Bernoulli

Messaggioda feddy » 13/10/2019, 16:12

ciao arnett, grazie per l'intervento. Quello che dici in effetti è vero. Purtroppo non conoscevo nemmeno tale risultato delle tre serie di Kolmogorov. Scegliendo $0<K<\frac{1}{n}$ dovrei riuscire a verificare e tre condizioni. Ad ogni modo mi scuso, ma ho dimenticato di specificare che le v.a. sono i.i.d !
Avatar utente
feddy
Moderatore
Moderatore
 
Messaggio: 2588 di 5934
Iscritto il: 26/06/2016, 00:25
Località: SISSA

Re: Convergenza quasi certa per una serie di Bernoulli

Messaggioda feddy » 13/10/2019, 16:45

Avevo iniziato a controllare le tre ipotesi con $K \leq \frac{1}{n}$ ma ho visto che le hai già controllate te, ti ringrazio! Con $0<K< \frac{1}{n}$ cambia solo che nelle sommatorie vado da 1 a $\frac{1}{K}$. Nella terza serie (quella con la varianza), mi risultava $\sum_{n}^{ceil(\frac{1}{K})} \frac{1}{n^2} \leq \sum_{n \in \mathbb{N}} \frac{1}{n^2} $ da cui la tesi.


Ora però non saprei come giustificare la convergenza a $0$ in $L^1$, poiché maggiorando la sommatoria non riesco a cavare fuori nulla
Avatar utente
feddy
Moderatore
Moderatore
 
Messaggio: 2589 di 5934
Iscritto il: 26/06/2016, 00:25
Località: SISSA

Re: Convergenza quasi certa per una serie di Bernoulli

Messaggioda feddy » 13/10/2019, 17:01

Qui sopra intendevi dire che non sei certo che converga a $0$ in $L^1$, vero? :)
Avatar utente
feddy
Moderatore
Moderatore
 
Messaggio: 2590 di 5934
Iscritto il: 26/06/2016, 00:25
Località: SISSA

Re: Convergenza quasi certa per una serie di Bernoulli

Messaggioda feddy » 13/10/2019, 17:54

Per l'uniforme integrabilità, devo mostrare che dato $\varepsilon >0$ posso sempre trovare $K\geq 0$ tale che $E[|Y_n| 1_{|Y_n| \geq K}] < \varepsilon$

Ma $|Y_n|=\frac{1}{n}$, quindi se scelgo $K> \frac{1}{n}$, allora ho che $E[|Y_n|1_{|Y_n| \geq K}] =0 < \varepsilon$
Ultima modifica di feddy il 13/10/2019, 17:57, modificato 1 volta in totale.
Avatar utente
feddy
Moderatore
Moderatore
 
Messaggio: 2591 di 5934
Iscritto il: 26/06/2016, 00:25
Località: SISSA

Re: Convergenza quasi certa per una serie di Bernoulli

Messaggioda feddy » 13/10/2019, 17:57

Ups questo va mostrato per la somma però
Avatar utente
feddy
Moderatore
Moderatore
 
Messaggio: 2592 di 5934
Iscritto il: 26/06/2016, 00:25
Località: SISSA

Re: Convergenza quasi certa per una serie di Bernoulli

Messaggioda feddy » 15/10/2019, 17:36

Update: ho "scoperto" che questo esercizio va risolto utilizzando il seguente teorema (cui dobbiamo acnora arrivare tra l'altro), ecco il motivo per cui non capivo dove andare a parare.

Sia $(X_n)_n$ una martingala w.r.t $(F_n)_n$ e sia $F_{\infty}= \sigma(F_n: n \in NN) $ Allora sono equivalenti
i) La martingala $(X_n)_n$ è uniformemente integrabile
ii) Esiste una variable aleatoria $F_{\infty}$ misurabile $X$ tale che $X_n \rarr X$ q.c. e in $L^1$



La domanda ora è: nel testo dell'esercizio, chi è la martingala? Quello che mi verrebbe naturale dire è che sia $X_n=\sum_{k=1}^{n} \frac{X_k}{k}$ con la filtrazione "ovvia" data da $F_n= \sigma(X_1,\ldots,X_n)$.

L'uniforme integrabilità di quest'ultima però ancora non mi viene: dovrei mostrare che $\text{sup}_{n} E[|X_n|] < \infty$, ma purtroppo maggiorando non vado da nessuno parte poiché

$E[|X_n|] =E[|\sum_{k=1}^{n} \frac{X_k}{k}|] \leq sum_{k=1}^{n} \frac{1}{k} E[|X_k|] = \sum_{k=1}^n \frac{1}{k} \cdot 1 \rarr_{n \rarr + infty} + \infty$

Idee?
Avatar utente
feddy
Moderatore
Moderatore
 
Messaggio: 2605 di 5934
Iscritto il: 26/06/2016, 00:25
Località: SISSA

Prossimo

Torna a Statistica e probabilità

Chi c’è in linea

Visitano il forum: Nessuno e 1 ospite